LSAT CR2

This topic has expert replies
Junior | Next Rank: 30 Posts
Posts: 22
Joined: Fri Mar 25, 2011 3:25 am
Thanked: 1 times

LSAT CR2

by abhishekpandey2002 » Mon Apr 04, 2011 10:26 pm
Contrary to the statements of labor leaders, the central economic problem facing America today is not the distribution of wealth. It is productivity. With the productivity of U.S. industry stagnant, or even declining slightly, the economic pie is no longer growing. Labor leaders, of course, point to what they consider an unfair distribution of the slices of pie to justify their demands for further increases in wages and benefits. And in the past, when the pie was still growing, management could afford to acquiesce. No longer. Until productivity resumes its growth, there can be no justification for further increases in the compensation of workers.
Which of the following statements by a labor leader focuses on the logical weakness in the argument above?
(A) Although the economic pie is no longer growing, the portion of the pie allocated to American workers remains unjustly small.
(B) If management fails to accommodate the demands of workers, labor leaders will be forced to call strikes that will cripple the operation of industry.
(C) Although productivity is stagnant, the U.S. population is growing, so that the absolute size of the economic pie continues to grow as well.
(D) As a labor leader, I can be concerned only with the needs of working people, not with the problems faced by management.
(E) The stagnation of U.S. industry has been caused largely by factors-such as foreign competition-beyond the control of American workers.

Junior | Next Rank: 30 Posts
Posts: 22
Joined: Fri Mar 25, 2011 3:25 am
Thanked: 1 times

by abhishekpandey2002 » Mon Apr 04, 2011 10:27 pm
I am confused between A and C. Can anybody please explain...

Legendary Member
Posts: 586
Joined: Tue Jan 19, 2010 4:38 am
Thanked: 31 times
Followed by:5 members
GMAT Score:730

by rohu27 » Tue Apr 05, 2011 1:49 am
IMO C.

important thng to note between options A and C is A says economic pie is stagnant where as C says its still growing.
Now look at the premise,
And in the past, when the pie was still growing, management could afford to acquiesce. No longer. Until productivity resumes its growth, there can be no justification for further increases in the compensation of workers.
the above is obviosly weakend by C, as pie is still growing, demands are justifed.

Junior | Next Rank: 30 Posts
Posts: 22
Joined: Fri Mar 25, 2011 3:25 am
Thanked: 1 times

by abhishekpandey2002 » Tue Apr 05, 2011 3:22 am
OA: A

Legendary Member
Posts: 586
Joined: Tue Jan 19, 2010 4:38 am
Thanked: 31 times
Followed by:5 members
GMAT Score:730

Legendary Member
Posts: 2330
Joined: Fri Jan 15, 2010 5:14 am
Thanked: 56 times
Followed by:26 members

by mundasingh123 » Wed Apr 06, 2011 12:51 am
rohu27 wrote:Stacey has a nice explanation here:

https://www.beatthegmat.com/wealth-distr ... 31809.html
Not happy with the Stacys OE.
(A) Although the economic pie is no longer growing, the portion of the pie allocated to American workers remains unjustly small.
The portion of the pie allocated to American workers remaining unjustly small is why the Workers have asked for an increase in their wages in the argument.
I Seek Explanations Not Answers

User avatar
GMAT Instructor
Posts: 905
Joined: Sun Sep 12, 2010 1:38 am
Thanked: 378 times
Followed by:123 members
GMAT Score:760

by Geva@EconomistGMAT » Wed Apr 06, 2011 2:10 am
mundasingh123 wrote:
rohu27 wrote:Stacey has a nice explanation here:

https://www.beatthegmat.com/wealth-distr ... 31809.html
Not happy with the Stacys OE.
(A) Although the economic pie is no longer growing, the portion of the pie allocated to American workers remains unjustly small.
The portion of the pie allocated to American workers remaining unjustly small is why the Workers have asked for an increase in their wages in the argument.
Precicesly. Note that the answer choices need to be counter arguments by the labor leaders to the argument above. You're trying to weaken the argument as to why the problem is productivity - or, in other words, you are trying to make the labor leaders' case that the pie, regardless of its size, is still unjustly divided.
Geva
Senior Instructor
Master GMAT
1-888-780-GMAT
https://www.mastergmat.com

Legendary Member
Posts: 2330
Joined: Fri Jan 15, 2010 5:14 am
Thanked: 56 times
Followed by:26 members

by mundasingh123 » Wed Apr 06, 2011 2:21 am
Geva@MasterGMAT wrote:
mundasingh123 wrote:
rohu27 wrote:Stacey has a nice explanation here:

https://www.beatthegmat.com/wealth-distr ... 31809.html
Not happy with the Stacys OE.
(A) Although the economic pie is no longer growing, the portion of the pie allocated to American workers remains unjustly small.
The portion of the pie allocated to American workers remaining unjustly small is why the Workers have asked for an increase in their wages in the argument.
Precicesly. Note that the answer choices need to be counter arguments by the labor leaders to the argument above. You're trying to weaken the argument as to why the problem is productivity - or, in other words, you are trying to make the labor leaders' case that the pie, regardless of its size, is still unjustly divided.
Hi Geva , A is the OA given by Stacy . I opted for E which is wrong . So U mean the argument is not about what the problem is ,but about whether the workers should be given an increase in benefits.
But the author never says that the pie is justly divided or the LL dont need an increase
I Seek Explanations Not Answers

User avatar
GMAT Instructor
Posts: 905
Joined: Sun Sep 12, 2010 1:38 am
Thanked: 378 times
Followed by:123 members
GMAT Score:760

by Geva@EconomistGMAT » Wed Apr 06, 2011 2:50 am
He does here: Until productivity resumes its growth, there can be no justification for further increases in the compensation of workers.

E is a type of claim that doesn't attack the logic of the argument: It's like saying "you're right, the economy is stagnating and that means that there's less money to go around, but it's not my fault that the economy is stagnating, so why are you punishing me?" - it's a semi-valid argument against the business perspective, but it doesn't do what the question asks you to do: find a statement that focuses on the logical weaknesses of the argument.
Geva
Senior Instructor
Master GMAT
1-888-780-GMAT
https://www.mastergmat.com

Legendary Member
Posts: 2330
Joined: Fri Jan 15, 2010 5:14 am
Thanked: 56 times
Followed by:26 members

by mundasingh123 » Wed Apr 06, 2011 3:05 am
Geva@MasterGMAT wrote:He does here: Until productivity resumes its growth, there can be no justification for further increases in the compensation of workers.

E is a type of claim that doesn't attack the logic of the argument: It's like saying "you're right, the economy is stagnating and that means that there's less money to go around, but it's not my fault that the economy is stagnating, so why are you punishing me?" - it's a semi-valid argument against the business perspective, but it doesn't do what the question asks you to do: find a statement that focuses on the logical weaknesses of the argument.
I thought the author was blaming the Labour for the economy's problems by saying productivity was to blame.and the Labours job was to say that it was not their fault.
but Geva
What else does "justification for further increases" / " to justify their demands for further increases in wages and benefits ". mean ?
Doesnt justification mean justifying the workers' demands .

Ok geva
1 final question .
Can you please explain the flaw in the argument a bit more clearly ?
I have failed to understand the flaw in the argument.
I Seek Explanations Not Answers